2018 AMC 10A Problems/Problem 22
Contents
Problem
Let and
be positive integers such that
,
,
, and
. Which of the following must be a divisor of
?
Solution 1
The GCD information tells us that divides
, both
and
divide
, both
and
divide
, and
divides
. Note that we have the prime factorizations:
Hence we have
for some positive integers
. Now if
divides
, then
would be at least
which is too large, hence
does not divide
. Similarly, if
divides
, then
would be at least
which is too large, so
does not divide
. Therefore,
where neither
nor
divide
. In other words,
is divisible only by primes that are at least
. The only possible value of
between
and
and which fits this criterion is
, so the answer is
.
Solution 2
We can say that and
'have'
, that
and
have
, and that
and
have
. Combining
and
yields
has (at a minimum)
, and thus
has
(and no more powers of
because otherwise
would be different). In addition,
has
, and thus
has
(similar to
, we see that
cannot have any other powers of
). We now assume the simplest scenario, where
and
. According to this base case, we have
. We want an extra factor between the two such that this number is between
and
, and this new factor cannot be divisible by
or
. Checking through, we see that
is the only one that works. Therefore the answer is
Solution by JohnHankock
Solution 3 (Better notation)
First off, note that ,
, and
are all of the form
. The prime factorizations are
,
and
, respectively. Now, let
and
be the number of times
and
go into
,respectively. Define
,
,
, and
similiarly. Now, translate the
s into the following:
.
(Unfinished) ~Rowechen Zhong
Solution 4 (Fastest)
Notice that , so
must be a multiple of
. The only answer choice that gives a value between
and
when multiplied by
is
. - mathleticguyyy + einstein
In the case where there can be 2 possible answers, we can do casework on
~Williamgolly
Solution 5
Since ,
and
for some positive integers
such that
and
are relatively prime.
Similarly , since , we have
and
with the same criteria. However, since
is not a multiple of
, we must contribute an extra
to
in order to make it a multiple of
. So,
is a multiple of three, and it is relatively prime to
.
Finally, , so using the same logic,
is a multiple of
and is relatively prime to
where
.
Since we can't really do anything with these messy expressions, we should try some sample cases of and
. Specifically, we let
or
, and see which one works.
First we let . Note that all of these values of
work for the first
expression because they are all not divisible by
.
Without the loss of generality, we let for all of our sample cases. We can also adjust the value of
in
, since there is no fixed value for
; there is only a bound.
So we try to make our bound satisfactory. We do so by letting
.
Testing our first case and
, we find that
. To simplify our work, we note that
, so
for all
is equal to
.
So now, we can easily find our values of :
We can clearly see that only is in the bound
. So,
must be a divisor of
, which is answer choice
.
-FIREDRAGONMATH16
Solution 6
The relationship of ,
,
, and
is shown in the above diagram.
.
,
,
Note that is not required to solve the problem.
Solution 7 (Easier version of Solution 1)
Just as in solution , we prime factorize
and
to observe that
Substituting these expressions for and
into the final given,
The greatest common divisor of these two numbers is already . If
is what we wish to multiply
by to obtain the gcd of these two numbers, then
. Testing the answer choices, only
works for
(in order for the compound inequality to hold). so our gcd is
, which means that
must divide
.
-Benedict T (countmath1)
Video Solution by Richard Rusczyk
https://artofproblemsolving.com/videos/amc/2018amc10a/467
~ dolphin7
Video Solution
~savannahsolver
Video Solution (Meta-Solving Technique)
https://youtu.be/GmUWIXXf_uk?t=1003
~ pi_is_3.14
See Also
2018 AMC 10A (Problems • Answer Key • Resources) | ||
Preceded by Problem 21 |
Followed by Problem 23 | |
1 • 2 • 3 • 4 • 5 • 6 • 7 • 8 • 9 • 10 • 11 • 12 • 13 • 14 • 15 • 16 • 17 • 18 • 19 • 20 • 21 • 22 • 23 • 24 • 25 | ||
All AMC 10 Problems and Solutions |
The problems on this page are copyrighted by the Mathematical Association of America's American Mathematics Competitions.